NIMO 2012 A1

Algebra Level 3

A number is called p u r p l e \textcolor{#69047E}{purple} if it can be expressed in the form 1 2 a 5 b \textcolor{#69047E}{\dfrac{1}{2^a5^b}} for positive integers a > b a>b . Find the sum of all p u r p l e \textcolor{#69047E}{purple} numbers.

1 12 \dfrac1{12} 1 10 \dfrac1{10} 1 11 \dfrac1{11} 1 13 \dfrac1{13} 1 9 \dfrac19

This section requires Javascript.
You are seeing this because something didn't load right. We suggest you, (a) try refreshing the page, (b) enabling javascript if it is disabled on your browser and, finally, (c) loading the non-javascript version of this page . We're sorry about the hassle.

2 solutions

1 < b < a < 1 2 a 5 b = b = 1 1 5 b a = b + 1 1 2 a = b = 1 1 5 b 1 2 b + 1 1 1 1 2 = b = 1 1 2 b 5 b = b = 1 1 1 0 b = 1 10 1 1 1 10 = 1 10 10 9 = 1 9 \begin{aligned} \sum_{1<b<a<\infty} \frac 1{2^a5^b} & = \sum_{b=1}^\infty \frac 1{5^b} \sum_{a=b+1}^\infty \frac 1{2^a} \\ & = \sum_{b=1}^\infty \frac 1{5^b} \cdot \frac 1{2^{b+1}} \cdot \frac 1{1-\frac 12} \\ & = \sum_{b=1}^\infty \frac 1{2^b5^b} \\ & = \sum_{b=1}^\infty \frac 1{10^b} \\ & = \frac 1{10} \cdot \frac 1{1-\frac 1{10}} \\ & = \frac 1{10} \cdot \frac {10}9 \\ & = \boxed{\frac 19} \end{aligned}

@Adhiraj Dutta , what about this:

Instead of 1 2 a 5 b \frac{1}{2^a5^b} , what about:

1 2 b 5 a \frac{1}{2^b5^a} and call them yellow numbers (Idea for next problem)

Log in to reply

Check if it converges or is reducible like this problem.

Adhiraj Dutta - 1 year ago

Log in to reply

Quick question: does a = b a = b ?

Log in to reply

@A Former Brilliant Member a = b a=b will result in an infinite GP.

Adhiraj Dutta - 1 year ago

Log in to reply

@Adhiraj Dutta 1 2 n 5 n \frac{1}{2^n5^n} = = 10 9 \frac{10}{9}

Don't set a problem which you don't understand.

Chew-Seong Cheong - 1 year ago

Log in to reply

I'm not setting a problem - it's just an idea...

Log in to reply

@A Former Brilliant Member Your this problem is wrong. I get x = 55 x=-55 and y = 1 y=-1 from the last two equations but when I substitute in the first i can't get 1 and 4.

Chew-Seong Cheong - 1 year ago

Log in to reply

@Chew-Seong Cheong Yes Chew sir, I get the same

Mahdi Raza - 1 year ago

@Chew-Seong Cheong Couldn't you post it in my Report Room...

i i ' t h ^{th} term of the sequence is 1 2 i + 3 ( 1 1 5 i ) = 1 8 ( 1 2 i 1 1 0 i ) \dfrac{1}{2^{i+3}}\left (1-\dfrac {1}{5^i}\right ) =\dfrac{1}{8}\left (\dfrac{1}{2^i}-\dfrac {1}{10^i}\right )

Therefore the required sum is 1 8 ( 1 1 1 2 1 1 1 10 ) = 1 9 \dfrac {1}{8}\left (\dfrac {1}{1-\frac{1}{2}}-\dfrac {1}{1-\frac{1}{10}}\right ) =\boxed {\dfrac{1}{9}} .

What do you mean terms???

0 pending reports

×

Problem Loading...

Note Loading...

Set Loading...